Skip to main content
Log in

On the Life Distribution Behavior of the Generalized Mixed \(\delta\)-Shock Models for the Multi-state Systems

  • Research Paper
  • Published:
Iranian Journal of Science and Technology, Transactions A: Science Aims and scope Submit manuscript

Abstract

In this paper, the life distribution behavior of a generalization of the mixed \(\delta\)-shock models in the multi-state systems is studied. In this model, the k out of interarrival times between two successive shocks with a magnitude less than \(\delta\) have a disaster result on the system which causes a complete failure. In addition to this event, another factor called the magnitude of shock causes the failure of the system, such that if the magnitude of a shock is greater than another critical threshold \(\gamma\), then the system fails. Such model create a multi-state system with a number of different states. The survival functions of the lifetime, the time spent by the system in a complete working state, and the total time spent by the system in partially working states are derived and the corresponding first two moments are also computed. An application in industry is analyzed to illustrate the proposed methodology. A simulation study is also presented to illustrate the behavior of the survival functions.

This is a preview of subscription content, log in via an institution to check access.

Access this article

Price excludes VAT (USA)
Tax calculation will be finalised during checkout.

Instant access to the full article PDF.

Institutional subscriptions

Fig. 1
Fig. 2
Fig. 3

Similar content being viewed by others

References

Download references

Author information

Authors and Affiliations

Authors

Corresponding author

Correspondence to A. R. Nematollahi.

Appendix

Appendix

Proof of Theorem 1

By the definition of T, we have

$$\begin{aligned} P(T>t)=P\left( \sum _{i=1}^{N}X_{i}>t\right) =\sum _{n=1}^{\infty }P\left( \sum _{i=1}^{N}X_{i}>t,N=n\right) . \end{aligned}$$

Let

$$\begin{aligned} \sum _{n=1}^{\infty }P\left( \sum _{i=1}^{N}X_{i}>t,N=n\right) =A+B+C+D, \end{aligned}$$

for \(n=1\) we have

$$\begin{aligned}&P(X_1>t,X_1<\delta ,Z_1>\gamma )+P(X_1>t,X_1>\delta ,Z_1>\gamma )\\&\quad =P(X_1>t,Z_1>\gamma )\\ \end{aligned}$$

and for \(n>1\)

$$\begin{aligned}&A=\sum _{n=k}^{\infty }\left( {\begin{array}{c}n-1\\ k-1\end{array}}\right) P\left( \sum _{i=1}^{n}X_{i}>t,X_{1}>\delta ,Z_{1}<\gamma ,\ldots ,X_{n-k}>\delta ,Z_{n-k}<\gamma ,\right. \\&\left. X_{n-k+1}<\delta ,Z_{n-k+1}<\gamma ,\ldots ,X_{n-1}<\delta ,Z_{n-1}<\gamma ,X_{n}<\delta \right) , \\&B=\sum _{n=k}^{\infty }\left( {\begin{array}{c}n-1\\ k-1\end{array}}\right) P\left( \sum _{i=1}^{n}X_{i}>t,X_{1}>\delta ,Z_{1}<\gamma ,\ldots ,X_{n-k}>\delta ,Z_{n-k}<\gamma ,\right. \\&\left. X_{n-k+1}<\delta ,Z_{n-k+1}<\gamma ,\ldots ,X_{n-1}<\delta ,Z_{n-1}<\gamma ,X_{n}>\delta ,Z_{n}>\gamma \right) , \\&C=\sum _{n=2}^{\infty }\sum _{j=0}^{k-2}\left( {\begin{array}{c}n-1\\ j\end{array}}\right) P\left( \sum _{i=1}^{n}X_{i}>t,X_{1}>\delta ,Z_{1}<\gamma ,\ldots ,X_{n-j-1}>\delta ,\right. \\&\left. Z_{n-j-1}<\gamma ,X_{n-j}<\delta ,Z_{n-j}<\gamma ,\ldots ,X_{n-1}<\delta ,Z_{n-1}<\gamma ,X_{n}>\delta ,Z_{n}>\gamma \right) , \\&D=\sum _{n=2}^{\infty }\sum _{j=0}^{k-2}\left( {\begin{array}{c}n-1\\ j\end{array}}\right) P\left( \sum _{i=1}^{n}X_{i}>t,X_{1}>\delta ,Z_{1}<\gamma ,\ldots ,X_{n-j-1}>\delta , \right. \\&\left. Z_{n-j-1}<\gamma ,X_{n-j}<\delta ,Z_{n-j}<\gamma ,\ldots ,X_{n-1}<\delta ,Z_{n-1}<\gamma ,X_{n}<\delta ,Z_{n}>\gamma \right) . \end{aligned}$$

In the following, we evaluate A, B, C and D. First note that

$$\begin{aligned} A&= \sum _{n=k}^{\infty }\left( {\begin{array}{c}n-1\\ k-1\end{array}}\right) P\left( \sum _{i=0}^{n}X_{i}>t,X_{1}> \delta ,Z_{1}<\gamma ,\ldots ,X_{n-k}>\delta ,Z_{n-k}<\gamma , \right. \\&\left. X_{n-k+1}<\delta ,Z_{n-k+1}<\gamma ,\ldots ,X_{n-1}<\delta ,Z_{n-1}<\gamma ,X_{n}<\delta \right) \\& = \sum _{n=k}^{\infty }\left( {\begin{array}{c}n-1\\ k-1\end{array}}\right) P\left( \sum _{i=1}^{n-1}X_{i}>t-X_{n},X_{1}>\delta ,Z_{1}<\gamma ,\ldots ,X_{n-k}>\delta , \right. \\&\left. Z_{n-k}<\gamma ,X_{n-k+1}<\delta ,Z_{n-k+1}<\gamma ,\ldots ,X_{n-1}<\delta ,Z_{n-1}<\gamma ,X_{n}<\delta \right) \\&= \sum _{n=k}^{\infty }\left( {\begin{array}{c}n-1\\ k-1\end{array}}\right) \int _{0}^{\delta }P\left( \sum _{i=1}^{n-1}X_{i}>t-x,X_{1}>\delta ,Z_{1}<\gamma ,\ldots ,X_{n-k}>\delta ,\right. \\&\left. Z_{n-k}<\gamma ,X_{n-k+1}<\delta ,Z_{n-k+1}<\gamma ,\ldots ,X_{n-1}<\delta ,Z_{n-1}<\gamma \right) dF(x) \\&= \sum _{n=k}^{\infty }\left( {\begin{array}{c}n-1\\ k-1\end{array}}\right) P(X_{1}>\delta ,Z_{1}<\gamma ,\ldots ,X_{n-k}>\delta ,Z_{n-k}<\gamma ,X_{n-k+1}<\delta , \\&Z_{n-k+1}<\gamma ,\ldots ,X_{n-1}<\delta ,Z_{n-1}<\gamma ) \int _{0}^{\delta }P\left( \sum _{i=1}^{n-1}X_{i}>t-x|X_{1}>\delta , \right. \\&\left. Z_{1}<\gamma ,\ldots ,X_{n-k}>\delta ,Z_{n-k}<\gamma ,X_{n-k+1}\right. \\&\left.<\delta ,Z_{n-k+1}<\gamma ,\ldots ,X_{n-1}<\delta ,Z_{n-1}<\gamma \right) dF(x). \end{aligned}$$

Thus

$$\begin{aligned} A=\sum _{n=k}^{\infty }\left( {\begin{array}{c}n-1\\ k-1\end{array}}\right) p_{1}^{n-k}p_{2}^{k-1}\int _{0}^{\delta }P(S_{n-k}^{*}+S_{k-1}^{**}>t-x)dF(x), \end{aligned}$$

and

$$\begin{aligned} B& = \sum _{n=k}^{\infty }\left( {\begin{array}{c}n-1\\ k-1\end{array}}\right) P\left( \sum _{i=1}^{n}X_{i}>t,X_{1}> \delta ,Z_{1}<\gamma ,\ldots ,X_{n-k}>\delta ,Z_{n-k}<\gamma , \right. \\&\left. X_{n-k+1}<\delta ,Z_{n-k+1}<\gamma ,\ldots ,X_{n-1}<\delta ,Z_{n-1}<\gamma ,X_{n}>\delta ,Z_{n}>\gamma \right) \\& = \sum _{n=k}^{\infty }\left( {\begin{array}{c}n-1\\ k-1\end{array}}\right) P\left( \sum _{i=1}^{n}X_{i}>t,X_{1}>\delta ,Z_{1}<\gamma ,\ldots ,X_{n-k}>\delta ,Z_{n-k}<\gamma , \right. \\&\left. X_{n-k+1}<\delta ,Z_{n-k+1}<\gamma ,\ldots ,X_{n-1}<\delta ,Z_{n-1}<\gamma ,X_{n}>\delta \right) \\&-\sum _{n=k}^{\infty }\left( {\begin{array}{c}n-1\\ k-1\end{array}}\right) P\left( \sum _{i=1}^{n}X_{i}>t,X_{1}>\delta ,Z_{1}<\gamma ,\ldots ,X_{n-k}>\delta ,Z_{n-k}<\gamma , \right. \\&\left. X_{n-k+1}<\delta ,Z_{n-k+1}<\gamma ,\ldots ,X_{n-1}<\delta ,Z_{n-1}<\gamma ,X_{n}>\delta ,Z_{n}<\gamma \right) . \end{aligned}$$

Thus

$$\begin{aligned} B&= \sum _{n=k}^{\infty }\left( {\begin{array}{c}n-1\\ k-1\end{array}}\right) p_1^{n-k}p_2^{k-1}\\&\int _{\delta }^{\infty }P(S_{n-k}^{*}+S_{k-1}^{**}>t-x)dF(x) \\&-\sum _{n=k}^{\infty }\left( {\begin{array}{c}n-1\\ k-1\end{array}}\right) p_1^{n-k+1}p_2^{k-1} P(S_{n-k+1}^{*}+S_{k-1}^{**}>t), \\ \end{aligned}$$

and similar to above, we have

$$\begin{aligned} C& = \sum _{n=2}^{\infty }\sum _{j=0}^{k-2}\left( {\begin{array}{c}n-1\\ j\end{array}}\right) p_{1}^{n-j-1}p_{2}^{j}\\&\int _{\delta }^{\infty }P(S_{n-j-1}^{*}+S_{j}^{**}>t-x)dF(x) \\&-\sum _{n=2}^{\infty }\sum _{j=0}^{k-2}\left( {\begin{array}{c}n-1\\ j\end{array}}\right) p_{1}^{n-j}p_{2}^{j}P(S_{n-j}^{*}+S_{j}^{**}>t), \end{aligned}$$

and

$$\begin{aligned} D& = \sum _{n=2}^{\infty }\sum _{j=0}^{k-2}\left( {\begin{array}{c}n-1\\ j\end{array}}\right) p_{1}^{n-j-1}p_{2}^{j}\\&\int _{0}^{\delta }P(S_{n-j-1}^{*}+S_{j}^{**}>t-x)dF(x) \\&-\sum _{n=2}^{\infty }\sum _{j=0}^{k-2}\left( {\begin{array}{c}n-1\\ j\end{array}}\right) p_{1}^{n-j-1}p_{2}^{j+1}P(S_{n-j-1}^{*}+S_{j+1}^{**}>t). \end{aligned}$$

Thus

$$\begin{aligned}&P(T>t) \\&\quad =P(X_1>t,Z_1>\gamma )\\&\qquad +\sum _{n=k}^{\infty }\left( {\begin{array}{c}n-1\\ k-1\end{array}}\right) p_1^{n-k}p_2^{k-1}\int _{0}^{ \infty }P(S_{n-k}^{*}+S_{k-1}^{**}>t-x)dF(x) \\&\qquad -\sum _{n=k}^{\infty }\left( {\begin{array}{c}n-1\\ k-1\end{array}}\right) p_1^{n-k+1}p_2^{k-1}P(S_{n-k+1}^{*}+S_{k-1}^{**}>t) \\&\qquad +\sum _{n=2}^{\infty }\sum _{j=0}^{k-2}\left( {\begin{array}{c}n-1\\ j\end{array}}\right) p_1^{n-j-1}p_2^{j} \int _{0}^{\infty }P(S_{n-j-1}^{*}+S_{j}^{**}>t-x)dF(x) \\&\qquad -\sum _{n=2}^{\infty }\sum _{j=0}^{k-2}\left( {\begin{array}{c}n-1\\ j\end{array}}\right) p_1^{n-j}p_2^{j}P(S_{n-j}^{*}+S_{j}^{**}>t) \\&\qquad -\sum _{n=2}^{\infty }\sum _{j=0}^{k-2}\left( {\begin{array}{c}n-1\\ j\end{array}}\right) p_1^{n-j-1}p_2^{j+1}P(S_{n-j-1}^{*}+S_{j+1}^{**}>t). \end{aligned}$$

\(\square\)

Proof of Theorem 2

We have

$$\begin{aligned} P(S>t)& = P\left( \sum _{i=1}^{M}X_{i}>t\right) =\sum _{m=1}^{\infty }P\left( \sum _{i=1}^{m}X_{i}>t,M=m\right) \\ \end{aligned}$$

For \(m=1\) we have

$$\begin{aligned}&P(X_1>t,X_1<\delta )+P(X_1>t,X_1>\delta ,Z_1>\gamma )\\&\quad =(P(X_1<\delta )-P(X_1<t))I_{[0,\delta )}(t)+P(X_1>\max (t,\delta ),Z_1>\gamma ) \end{aligned}$$

and for \(m>1\) we have

$$\begin{aligned}&= \sum _{m=2}^{\infty }P\left( \sum _{i=1}^{m}X_{i}>t,X_{1}>\delta ,Z_{1}<\gamma ,\ldots ,X_{m-1}>\delta ,Z_{m-1}<\gamma ,X_{m}<\delta \right) \\&\quad +\sum _{m=2}^{\infty }P\left( \sum _{i=1}^{m}X_{i}>t,X_{1}>\delta ,Z_{1}<\gamma ,\ldots ,X_{m-1}>\delta ,Z_{m-1}<\gamma ,\right. \\&\quad \left. X_{m}>\delta ,Z_{m}>\gamma \right) \\&= \sum _{m=2}^{\infty }p_1^{m-1}\int _{0}^{\infty }P(S_{m-1}^{*}>t-x)dF(x)-\sum _{m=2}^{\infty }p_1^{m}P(S_{m}^{*}>t). \end{aligned}$$

Proof of Lemma 1

Proof of a) is clear. For b) note that

$$\begin{aligned} P(N-M=a|N>M)=\dfrac{P(N-M=a,N>M)}{P(N>M)}, \end{aligned}$$

by part a,

$$\begin{aligned} P(N-M& = a,N>M)=\sum _{m=1}^{\infty }P(N=m+a,M=m) \\&= \sum _{m=1}^{\infty }\left( {\begin{array}{c}a-1\\ k-2\end{array}}\right) p_{2}^{k-1}p_{1}^{m+a-k}(p_{3}+p_{4}) \\&+\sum _{m=1}^{\infty }\sum _{j=0}^{k-3}\left( {\begin{array}{c}a-1\\ j\end{array}}\right) p_{2}^{j+1}p_{1}^{m+a-j-2}(p_{4}+p_{5}) \\& = \dfrac{p_{2}}{1-p_{1}}\bigg [\left( {\begin{array}{c}a-1\\ k-2\end{array}}\right) p_{2}^{k-2}p_{1}^{a-k+1}(p_{3}+p_{4}) \\&+\sum _{j=0}^{k-3}\left( {\begin{array}{c}a-1\\ j\end{array}}\right) p_{2}^{j}p_{1}^{a-j-1}(p_{4}+p_{5})\bigg ], \end{aligned}$$

and

$$\begin{aligned}&P(N>M)\\&\quad =\sum _{m=1}^{\infty }\sum _{n=m+k-1}^{\infty }\left( {\begin{array}{c}n-m-1\\ k-2\end{array}}\right) p_{2}^{k-1}p_{1}^{n-k}(p_{3}+p_{4}) \\&\qquad +\sum _{m=1}^{\infty }\sum _{n=m+j+1}^{\infty }\sum _{j=0}^{k-3}\left( {\begin{array}{c}n-m-1\\ j\end{array}}\right) p_{2}^{j+1}p_{1}^{n-j-2}(p_{4}+p_{5}) \\&\quad =\dfrac{p_{2}}{1-p_{1}}\bigg (\dfrac{p_{2}^{k-2}}{(1-p_{1})^{k-2}}+1-\dfrac{ p_{2}^{k-2}}{(1-p_{1})^{k-2}}\bigg ) \\&\quad =\dfrac{p_{2}}{1-p_{1}}. \end{aligned}$$

Thus

$$\begin{aligned}&P(N-M=a|N>M) \\&\quad =\left( {\begin{array}{c}a-1\\ k-2\end{array}}\right) p_{2}^{k-2}p_{1}^{a-k+1}(p_{3}+p_{4}) \\&\qquad +\sum _{j=0}^{k-3}\left( {\begin{array}{c}a-1\\ j\end{array}}\right) p_{2}^{j}p_{1}^{a-j-1}(p_{4}+p_{5}). \end{aligned}$$

It is the distribution of N for \(k-1\) instead of k. \(\square\)

Proof of Theorem 3

We have

$$\begin{aligned}&P(T-S=0)\\&\quad =P(T-S=0,N=M)+P(T-S=0,N>M)\\&\quad =P(T-S=0,N=M) \\&\quad = P(T-S=0|N=M)P(N=M)=P(N=M)\\&\quad =\sum _{n=1}^{\infty }P(N=n,M=n)\\&\quad =\sum _{n=1}^{\infty }p_1^{n-1}(p_4+p_5)\\&\quad =\dfrac{p_4+p_5}{1-p_1}, \end{aligned}$$

For \(t>0,\)

$$\begin{aligned} P(T-S>t)& = P\left( \sum _{i=1}^{N}X_{i}-\sum _{i=1}^{M}X_{i}>t|N>M\right) P(N>M) \\& = P\left( \sum _{i=M+1}^{N}X_{i}>t|N>M\right) P(N>M) \\& = P\left( \sum _{i=1}^{N-M}X_{i}>t|N>M\right) P(N>M) \\& = P(N>M)P\left( \sum _{i=1}^{N^{*}}X_{i}>t\right) , \end{aligned}$$

where \(P(\sum _{i=1}^{N^{*}}X_{i}>t)\) is similar to that of Theorem 1 for \(k-1\) instead of k.

Proof of Proposition 1

(a) We have

$$\begin{aligned} E(T)& = E\left( \sum _{i=1}^{N}X_{i}\right) =\sum _{n=1}^{\infty }E\left( \left( \sum _{i=1}^{n}X_{i}\right) |N=n\right) P(N=n) \\& = \sum _{n=1}^{\infty }\sum _{i=1}^{n}E(X_{i}|N=n)P(N=n). \end{aligned}$$

From the definition of N, we find

$$\begin{aligned}&E(X_{i}|N=n)\\&\quad =\left\{ \begin{array}{ll} E_{1}, &{} i=1,\ldots ,n-k \\ E_{2}, &{} i=n-k+1,\ldots ,n-1 \\ \dfrac{p_{3}}{p_{3}+p_{4}}E_{3}+\dfrac{p_{4}}{p_{3}+p_{4}}E_{4}, &{} i=n \end{array} \right. \end{aligned}$$

and for \(j=0,1,\ldots ,k-2,\)

$$\begin{aligned}&E(X_{i}|N=n)\\&\quad =\left\{ \begin{array}{ll} E_{1}, &{} i=1,\ldots ,n-j-1 \\ E_{2}, &{} i=n-j,\ldots ,n-1 \\ \dfrac{p_{4}}{p_{4}+p_{5}}E_{4}+\dfrac{p_{5}}{p_{4}+p_{5}}E_{5}, &{} i=n. \end{array} \right. \end{aligned}$$

Thus

$$\begin{aligned}&\sum _{n=1}^{\infty }\sum _{i=1}^{n}E(X_{i}|N =n)P(N=n) \\&\quad =A_{1}\times \left[ E(N-k)E_{1}+(k-1)E_{2}\right. \\&\qquad \left. +\dfrac{p_{3}}{p_{3}+p_{4}} E_{3}+\dfrac{p_{4}}{p_{3}+p_{4}}E_{4}\right] \\&\qquad +\sum _{j=2}^{k}A_{j}\times \left[ E(N-(j-1))E_{1}+(j-2)E_{2}+\dfrac{p_{5} }{p_{4}+p_{5}}E_{5}\right. \\&\qquad \left. +\dfrac{p_{4}}{p_{4}+p_{5}}E_{4}\right] . \end{aligned}$$

(b) According to Parvardeh and Balakrishnan (2015), and by using the Wald identity, we have

$$\begin{aligned} E\left( \sum _{i=1}^{M}X_{i}\right) =E(M)E(X_{1}). \end{aligned}$$

(c)

$$\begin{aligned}&E(T-S)\\&\quad =E\left( \sum _{i=1}^{N}X_{i}-\sum _{i=1}^{M}X_{i}\right) \\&\quad =E\left( \sum _{i=1}^{N}X_{i}- \sum _{i=1}^{M}X_{i}|N=M\right) P(N=M) \\&\qquad +E\left( \sum _{i=1}^{N}X_{i}-\sum _{i=1}^{M}X_{i}|N>M\right) P(N>M) \\&\quad =E\left( \sum _{i=1}^{N}X_{i}-\sum _{i=1}^{M}X_{i}|N>M\right) P(N>M) \\&\quad =E\left( \sum _{i=M+1}^{N}X_{i}|N>M\right) P(N>M) \\&\quad =E\left( \sum _{i=1}^{N^{*}}X_{i}\right) P(N>M), \end{aligned}$$

where \(E(\sum _{i=1}^{N^{*}}X_{i})\) is similar to E(T) for \(k-1\) instead of k. \(\square\)

Proof of Proposition 2

We prove \(E(T^2)\) only. \(E(S^2)\) and \(E(T-S)^2\) are similar.

$$\begin{aligned} E(T^{2})=E\left( \sum _{i=1}^{N}X_{i}\right) ^{2}=\sum _{n=1}^{\infty }E\left( \left( \sum _{i=1}^{n}X_{i}\right) ^{2}|N=n\right) P(N=n), \end{aligned}$$

where

$$\begin{aligned} E\left( \left( \sum _{i=1}^{n}X_{i}\right) ^{2}|N=n\right)& = \sum _{i=1}^{n}E(X_{i}^{2}|N=n)\\&+2\sum \sum _{1\le i<l\le n}E(X_{i}X_{l}|N=n). \end{aligned}$$

From the definition of N, we find

$$\begin{aligned} E(X_{i}^{2}|N=n)=\left\{ \begin{array}{ll} E_{1}^{^{\prime }}, &{} i=1,\ldots ,n-k \\ E_{2}^{^{\prime }}, &{} i=n-k+1,\ldots ,n-1 \\ \dfrac{p_{3}}{p_{3}+p_{4}}E_{3}^{^{\prime }}+\dfrac{p_{4}}{p_{3}+p_{4}} E_{4}^{^{\prime }}, &{} i=n \end{array} \right. \end{aligned}$$

and for \(j=0,1,\ldots ,k-2,\)

$$\begin{aligned} E(X_{i}^{2}|N=n)=\left\{ \begin{array}{ll} E_{1}^{^{\prime }}, &{} i=1,\ldots ,n-j-1 \\ E_{2}^{^{\prime }}, &{} i=n-j,\ldots ,n-1 \\ \dfrac{p_{4}}{p_{4}+p_{5}}E_{4}^{^{\prime }}+\dfrac{p_{5}}{p_{4}+p_{5}} E_{5}^{^{\prime }}, &{} i=n. \end{array} \right. \end{aligned}$$

Thus

$$\begin{aligned}&\sum _{n=1}^{\infty }\sum _{i=1}^{n}E(X_{i}^{2}|N = n)P(N=n) \\&\quad =A_{1}\times \left[ E(N-k)E_{1}^{^{\prime }}+(k-1)E_{2}^{^{\prime }}+ \dfrac{p_{3}}{p_{3}+p_{4}}E_{3}^{^{\prime }}+\dfrac{p_{4}}{p_{3}+p_{4}} E_{4}^{^{\prime }}\right] \\&\qquad +\sum _{j=2}^{k}A_{j}\times \left[ E(N-(j-1))E_{1}^{^{\prime }}+(j-2)E_{2}^{^{\prime }}+\dfrac{p_{5}}{p_{4}+p_{5}}E_{5}^{^{\prime }}\right. \\&\qquad \left. + \dfrac{p_{4}}{p_{4}+p_{5}}E_{4}^{^{\prime }}\right] . \end{aligned}$$

and

$$\begin{aligned}&E(X_{i}X_{l}|N=n) \\&\quad =\left\{ \begin{array}{ll} E_{1}^{2}, &{} 1\le i<l\le n-k \\ E_{2}^{2}, &{} n-k+1\le i<l\le n-1, \\ E_{1}E_{2}, &{} 1\le i\le n-k,n-k+1\le l\le n-1 \\ \dfrac{p_{3}}{p_{3}+p_{4}}E_{1}E_{3}+\dfrac{p_{4}}{p_{3}+p_{4}}E_{1}E_{4}, &{} 1\le i\le n-k,l=n \\ \dfrac{p_{3}}{p_{3}+p_{4}}E_{2}E_{3}+\dfrac{p_{4}}{p_{3}+p_{4}}E_{2}E_{4}, &{} n-k+1\le i\le n-1<l=n \end{array} \right. \end{aligned}$$

For \(j=0\) we have

$$\begin{aligned} E(X_{i}X_{l}|N=n)= \\ \left\{ \begin{array}{ll} E_{1}^{2}, &{} 1\le i<l\le n-1 \\ \dfrac{p_{4}}{p_{4}+p_{5}}E_{1}E_{4}+\dfrac{p_{5}}{p_{4}+p_{5}}E_{1}E_{5}, &{} 1\le i<l=n, \end{array} \right. \end{aligned}$$

and for \(j=1,2,\ldots ,k-2\) we have

$$\begin{aligned}&E(X_{i}X_{l}|N=n) \\&\quad =\left\{ \begin{array}{ll} E_{1}^{2}, &{} 1\le i<l\le n-j-1 \\ E_{1}E_{2}, &{} 1\le i\le n-j-1,n-j\le l\le n-1 \\ E_{2}^{2}, &{} n-j\le i<l\le n-1 \\ \dfrac{p_{4}}{p_{4}+p_{5}}E_{1}E_{4}+\dfrac{p_{5}}{p_{4}+p_{5}}E_{1}E_{5}, &{} 1\le i\le n-j-1,l=n \\ \dfrac{p_{4}}{p_{4}+p_{5}}E_{2}E_{4}+\dfrac{p_{5}}{p_{4}+p_{5}}E_{2}E_{5}, &{} n-j\le i\le n-1,l=n. \end{array} \right. \end{aligned}$$

Then

$$\begin{aligned}&2\sum _{n=1}^{\infty }\sum \sum _{1\le i<l\le n}E(X_{i}X_{l}|N=n)P(N=n) \\&\quad =2\times \left[ A_{1}\times \left( E\left( \frac{(N-k)(N-k-1)}{2}\right) E_1^{2} +\frac{ (k-1)(k-2)}{2}E_2^{2} \right. \right. \\&\qquad \left. +(k-1)E(N-k)E_1E_2+E(N-k)\right. \\&\qquad \left. \left( \frac{p_3}{p_3+p_4}E_1E_3+\frac{p_4}{p_3+p_4} E_1E_4\right) \right. \\&\qquad \left. \left. +(k-1)\left( \frac{p_3}{p_3+p_4}E_2E_3 +\frac{p_4}{p_3+p_4}E_2E_4\right) \right) \right. \\&\qquad \left. +\sum _{j=2}^{k}A_{j} \left( \frac{E(N-j+1)(N-j)}{2}E_1^{2} +(j-2)E(N-j+1)E_1E_2 \right. \right. \\&\qquad \left. \left. +\frac{(j-2)(j-3)}{2}E_2^{2} +(j-2)\right. \right. \\&\qquad \left. \left. \times \left( \frac{p_4}{p_4+p_5}E_2E_4+ \frac{p_5}{p_4+p_5}E_2E_5\right) \right. \right. \\&\qquad \left. \left. +E(N-j+1)\times \left( \frac{p_4}{p_4+p_5}E_1E_4+\frac{p_5}{p_4+p_5}E_1E_5 \right) \right) \right] . \end{aligned}$$

In this part E(N) and \(E(N^{2})\) are calculated.

$$\begin{aligned} E(N)& = \sum _{n=k}^{\infty }n\left( {\begin{array}{c}n-1\\ k-1\end{array}}\right) p_{2}^{k-1}p_{1}^{n-k}(p_{3}+p_{4}) \\&+\sum _{n=1}^{\infty }\sum _{j=0}^{k-2}n\left( {\begin{array}{c}n-1\\ j\end{array}}\right) p_{2}^{j}p_{1}^{n-j-1}(p_{4}+p_{5}). \end{aligned}$$

Let

$$\begin{aligned} A^{1}& = \sum _{n=k}^{\infty }n\left( {\begin{array}{c}n-1\\ k-1\end{array}}\right) p_{2}^{k-1}p_{1}^{n-k}(p_{3}+p_{4}) \\ B^{1}& = \sum _{n=1}^{\infty }\sum _{j=0}^{k-2}n\left( {\begin{array}{c}n-1\\ j\end{array}}\right) p_{2}^{j}p_{1}^{n-j-1}(p_{4}+p_{5}). \end{aligned}$$

Then

$$\begin{aligned} A^{1}& = p_{2}^{k-1}(p_{3}+p_{4}).\sum _{n=k}^{\infty }k\left( {\begin{array}{c}n\\ k\end{array}}\right) p_{1}^{n-k}\\& = p_{2}^{k-1}(p_{3}+p_{4})k\frac{1}{(1-p_{1})^{k+1}} \\& = k\dfrac{p_{2}^{k-1}}{(1-p_{1})^{k}}, \end{aligned}$$

and

$$\begin{aligned} B^{1}& = \sum _{n=1}^{\infty }\sum _{j=0}^{k-2}n\left( {\begin{array}{c}n-1\\ j\end{array}}\right) p_{1}^{n-j-1}p_{2}^{j}(p_{4}+p_{5})\\& = (p_{4}+p_{5})\frac{\partial }{\partial p_{2}}\dfrac{\frac{p_{2}}{1-p_{1}}\left( 1-\left( \frac{p_{2}}{1-p_{1}}\right) ^{k-1}\right) }{ 1-p_{2}-p_{1}}. \end{aligned}$$

Then

$$\begin{aligned} E(N)=\dfrac{(1-p_{1})^{k}-p_{2}^{k}}{(1-p_{2}-p_{1})(1-p_{1})^{k}}, \end{aligned}$$

and

$$\begin{aligned} E(N^{2})& = \sum _{n=k}^{\infty }n^{2}\left( {\begin{array}{c}n-1\\ k-1\end{array}}\right) p_{2}^{k-1}p_{1}^{n-k}(p_{3}+p_{4}) \\&+\sum _{n=1}^{\infty }\sum _{j=0}^{k-2}n^{2}\left( {\begin{array}{c}n-1\\ j\end{array}}\right) p_{2}^{j}p_{1}^{n-j-1}(p_{4}+p_{5}). \end{aligned}$$

Define \(C^{1}\) and \(D^{1}\) as

$$\begin{aligned} C^{1}& = \sum _{n=k}^{\infty }n^{2}\left( {\begin{array}{c}n-1\\ k-1\end{array}}\right) p_{2}^{k-1}p_{1}^{n-k}(p_{3}+p_{4}), \\ D^{1}& = \sum _{n=1}^{\infty }\sum _{j=0}^{k-2}n^{2}\left( {\begin{array}{c}n-1\\ j\end{array}}\right) p_{2}^{j}p_{1}^{n-j-1}(p_{4}+p_{5}). \end{aligned}$$

Then

$$\begin{aligned} C^{1}=\frac{kp_{2}^{k-1}(p_{1}+k)}{(1-p_{1})^{k+1}}, \end{aligned}$$

and

$$\begin{aligned} D^{1}&= \dfrac{p_{4}+p_{5}}{1-p_{1}}\left[ \frac{\partial }{\partial p_{2}}\left( \dfrac{p_{2}-\frac{p_{2}^{k}}{(1-p_{1})^{k-1}}}{1-p_{2}-p_{1}}\right) \right. \\&\quad \left. +(p_{2}+p_{1}) \frac{\partial ^{2}}{\partial p_{2}^{2}}\left( \dfrac{p_{2}-\frac{p_{2}^{k}}{ (1-p_{1})^{k-1}}}{1-p_{2}-p_{1}}\right) \right] . \end{aligned}$$

\(\square\)

Proof of Lemma 2

We know

$$\begin{aligned} P(M=m|N>M)=\dfrac{P(M=m,N>M)}{P(N>M)}, \end{aligned}$$

and by using Lemma 1, we have

$$\begin{aligned} P(N>M)=\dfrac{p_{2}}{1-p_{1}}, \end{aligned}$$

and

$$\begin{aligned}&P(M=m,N>M) \\&\quad = \sum _{n=m+1}^{\infty }\left( {\begin{array}{c}n-m-1\\ k-2\end{array}}\right) p_{2}^{k-1}p_{1}^{n-k}(p_{3}+p_{4}) \\&\qquad +\sum _{n=m+1}^{\infty }\sum _{j=0}^{k-3}\left( {\begin{array}{c}n-m-1\\ j\end{array}}\right) p_{2}^{j+1}p_{1}^{n-j-2}(p_{4}+p_{5}) \\&\quad =p_{1}^{m-1}p_{2}. \end{aligned}$$

Then

$$\begin{aligned}&P(M=m|N>M)=\dfrac{p_{1}^{m-1}p_{2}}{\frac{p_{2}}{1-p_{1}}}\\&\quad =p_{1}^{m-1}(1-p_{1}),\quad m=1,2,\ldots \end{aligned}$$

\(\square\)

Rights and permissions

Reprints and permissions

About this article

Check for updates. Verify currency and authenticity via CrossMark

Cite this article

Lorvand, H., Poursaeed, M.H. & Nematollahi, A.R. On the Life Distribution Behavior of the Generalized Mixed \(\delta\)-Shock Models for the Multi-state Systems. Iran J Sci Technol Trans Sci 44, 839–850 (2020). https://doi.org/10.1007/s40995-020-00884-6

Download citation

  • Received:

  • Accepted:

  • Published:

  • Issue Date:

  • DOI: https://doi.org/10.1007/s40995-020-00884-6

Keywords

Navigation